Eigenvalues of irreducible matrix.












0












$begingroup$


$A in M_{3x3}(mathbb{R})$ is irreducible and non-negaive matrix with three eigenvalues $lambda_1 , lambda_2 , lambda_3$. Can it be true :




  1. $lambda_1 = lambda_2 = lambda_3= varrho (A)$

  2. $lambda_1 =i lambda_2 =-i lambda_3=3$

  3. $lambda_1=2 , lambda_2=-2 , lambda_3=1$

  4. $lambda_1=2 , lambda_2=1 , lambda_3=-1$


I think, that first can't be true, because it comes from Frobenius theorem.But I don't know what about the others. So,
1-No, 2-Yes, 3-No, 4-Yes.










share|cite|improve this question











$endgroup$








  • 1




    $begingroup$
    $det mathbf{A}=prod_{k=1}^{3} lambda_{k}$
    $endgroup$
    – dantopa
    Jan 22 at 23:15


















0












$begingroup$


$A in M_{3x3}(mathbb{R})$ is irreducible and non-negaive matrix with three eigenvalues $lambda_1 , lambda_2 , lambda_3$. Can it be true :




  1. $lambda_1 = lambda_2 = lambda_3= varrho (A)$

  2. $lambda_1 =i lambda_2 =-i lambda_3=3$

  3. $lambda_1=2 , lambda_2=-2 , lambda_3=1$

  4. $lambda_1=2 , lambda_2=1 , lambda_3=-1$


I think, that first can't be true, because it comes from Frobenius theorem.But I don't know what about the others. So,
1-No, 2-Yes, 3-No, 4-Yes.










share|cite|improve this question











$endgroup$








  • 1




    $begingroup$
    $det mathbf{A}=prod_{k=1}^{3} lambda_{k}$
    $endgroup$
    – dantopa
    Jan 22 at 23:15
















0












0








0





$begingroup$


$A in M_{3x3}(mathbb{R})$ is irreducible and non-negaive matrix with three eigenvalues $lambda_1 , lambda_2 , lambda_3$. Can it be true :




  1. $lambda_1 = lambda_2 = lambda_3= varrho (A)$

  2. $lambda_1 =i lambda_2 =-i lambda_3=3$

  3. $lambda_1=2 , lambda_2=-2 , lambda_3=1$

  4. $lambda_1=2 , lambda_2=1 , lambda_3=-1$


I think, that first can't be true, because it comes from Frobenius theorem.But I don't know what about the others. So,
1-No, 2-Yes, 3-No, 4-Yes.










share|cite|improve this question











$endgroup$




$A in M_{3x3}(mathbb{R})$ is irreducible and non-negaive matrix with three eigenvalues $lambda_1 , lambda_2 , lambda_3$. Can it be true :




  1. $lambda_1 = lambda_2 = lambda_3= varrho (A)$

  2. $lambda_1 =i lambda_2 =-i lambda_3=3$

  3. $lambda_1=2 , lambda_2=-2 , lambda_3=1$

  4. $lambda_1=2 , lambda_2=1 , lambda_3=-1$


I think, that first can't be true, because it comes from Frobenius theorem.But I don't know what about the others. So,
1-No, 2-Yes, 3-No, 4-Yes.







matrices






share|cite|improve this question















share|cite|improve this question













share|cite|improve this question




share|cite|improve this question








edited Jan 22 at 23:15







pawelK

















asked Jan 22 at 23:01









pawelKpawelK

527




527








  • 1




    $begingroup$
    $det mathbf{A}=prod_{k=1}^{3} lambda_{k}$
    $endgroup$
    – dantopa
    Jan 22 at 23:15
















  • 1




    $begingroup$
    $det mathbf{A}=prod_{k=1}^{3} lambda_{k}$
    $endgroup$
    – dantopa
    Jan 22 at 23:15










1




1




$begingroup$
$det mathbf{A}=prod_{k=1}^{3} lambda_{k}$
$endgroup$
– dantopa
Jan 22 at 23:15






$begingroup$
$det mathbf{A}=prod_{k=1}^{3} lambda_{k}$
$endgroup$
– dantopa
Jan 22 at 23:15












0






active

oldest

votes











Your Answer





StackExchange.ifUsing("editor", function () {
return StackExchange.using("mathjaxEditing", function () {
StackExchange.MarkdownEditor.creationCallbacks.add(function (editor, postfix) {
StackExchange.mathjaxEditing.prepareWmdForMathJax(editor, postfix, [["$", "$"], ["\\(","\\)"]]);
});
});
}, "mathjax-editing");

StackExchange.ready(function() {
var channelOptions = {
tags: "".split(" "),
id: "69"
};
initTagRenderer("".split(" "), "".split(" "), channelOptions);

StackExchange.using("externalEditor", function() {
// Have to fire editor after snippets, if snippets enabled
if (StackExchange.settings.snippets.snippetsEnabled) {
StackExchange.using("snippets", function() {
createEditor();
});
}
else {
createEditor();
}
});

function createEditor() {
StackExchange.prepareEditor({
heartbeatType: 'answer',
autoActivateHeartbeat: false,
convertImagesToLinks: true,
noModals: true,
showLowRepImageUploadWarning: true,
reputationToPostImages: 10,
bindNavPrevention: true,
postfix: "",
imageUploader: {
brandingHtml: "Powered by u003ca class="icon-imgur-white" href="https://imgur.com/"u003eu003c/au003e",
contentPolicyHtml: "User contributions licensed under u003ca href="https://creativecommons.org/licenses/by-sa/3.0/"u003ecc by-sa 3.0 with attribution requiredu003c/au003e u003ca href="https://stackoverflow.com/legal/content-policy"u003e(content policy)u003c/au003e",
allowUrls: true
},
noCode: true, onDemand: true,
discardSelector: ".discard-answer"
,immediatelyShowMarkdownHelp:true
});


}
});














draft saved

draft discarded


















StackExchange.ready(
function () {
StackExchange.openid.initPostLogin('.new-post-login', 'https%3a%2f%2fmath.stackexchange.com%2fquestions%2f3083837%2feigenvalues-of-irreducible-matrix%23new-answer', 'question_page');
}
);

Post as a guest















Required, but never shown

























0






active

oldest

votes








0






active

oldest

votes









active

oldest

votes






active

oldest

votes
















draft saved

draft discarded




















































Thanks for contributing an answer to Mathematics Stack Exchange!


  • Please be sure to answer the question. Provide details and share your research!

But avoid



  • Asking for help, clarification, or responding to other answers.

  • Making statements based on opinion; back them up with references or personal experience.


Use MathJax to format equations. MathJax reference.


To learn more, see our tips on writing great answers.




draft saved


draft discarded














StackExchange.ready(
function () {
StackExchange.openid.initPostLogin('.new-post-login', 'https%3a%2f%2fmath.stackexchange.com%2fquestions%2f3083837%2feigenvalues-of-irreducible-matrix%23new-answer', 'question_page');
}
);

Post as a guest















Required, but never shown





















































Required, but never shown














Required, but never shown












Required, but never shown







Required, but never shown

































Required, but never shown














Required, but never shown












Required, but never shown







Required, but never shown







Popular posts from this blog

Mario Kart Wii

What does “Dominus providebit” mean?

Antonio Litta Visconti Arese